Đến nội dung

Hình ảnh

không phải dễ

- - - - -

  • Please log in to reply
Chủ đề này có 4 trả lời

#1
slbadguy

slbadguy

    Hạ sĩ

  • Thành viên
  • 76 Bài viết
Tìm giá trị lớn nhất của biểu thức sau:
S=$ \dfrac{100-x_1}{100}+\dfrac{x_1-x_2}{x_1}+...+\dfrac{x_{n-1}-x_n}{x_{n-1}}$
với $ x_1,x_2,...,x_n \in N$ và $ 100 \geq x_1 \geq ... \geq x_n>0$

Bài viết đã được chỉnh sửa nội dung bởi slbadguy: 10-01-2008 - 19:49


#2
vd_tan

vd_tan

    Chuyên toán

  • Thành viên
  • 137 Bài viết
Post lời giải đi bạn

#3
Songohan

Songohan

    Trung sĩ

  • Thành viên
  • 181 Bài viết
Biết làm là chết liền bạn ơi, dạng bất đẳng thức trên tập rời rạc thế này khó giải lắm.
Cái này thực chất là bắt nguồn từ câu hỏi: một người tham dự chương trình ĐẤU TRƯỜNG 100 có thể nhận nhiều nhất là bao nhiêu tiền và phải chơi như thế nào?
Cái tổng chính là số tiền.

#4
y chi

y chi

    Trung sĩ

  • Thành viên
  • 140 Bài viết
Theo tôi vấn đề là tìm n.
Ta có $\dfrac{x_i-x_j}{x_i} \leq \sum\limits_{k=1}^{n} \dfrac{1}{x_{j+k}}$ TRong đó: $x_i=x_j+n$
THứ hai: $x_1=100 $thì khi đó $x_2 $tương tự $x_1$. Nên$ x_1=99$
Vậy max=$\dfrac{1}{2}+\dfrac{1}{3}+...+\dfrac{1}{100}$
ý chí là vũ khí mạnh nhất của bạn

#5
hoàngminh

hoàngminh

    Binh nhì

  • Thành viên
  • 10 Bài viết
uhm, bài viết này là bài viết đầu tiên cho ddth nè.


Cách giải của mình cho bài trên như sau:
Các bạn để ý rằng điều kiện 100 :sum $x_1$ :leq $x_2 $ (*) ... (*) $x_n$ > 0 không khác gì điều kiện $100 > x_1 > x_2 > ... > x_n > 0$ nên ta thay điều kiện ban đầu thành điều kiện $100 > x_1 > x_2 > ... > x_n > 0$.
Ta chọn 1 bộ số tự nhiên bất kì $(x_1, x_2, x_3,..., x_n)$ thỏa mãn $100 > x_1 > x_2 > ... > x_n > 0$
Coi $100$ là $x_0$ và $0$ là $x_{n+1}$.
Nếu tồn tại 1 số tự nhiên k thỏa mãn $x_i > k > x_{i+1}$ ( i = 0,1,2,...,n )
Ta có $S=n - (\dfrac {x_1}{x_0} +\dfrac{ x_2}{x_1} + ... + \dfrac{x_i}{x_{i-1}} + \dfrac{x_{i+1}}{x_i} + \dfrac{x_{i+2}}{x_{i+1}} + ... + \dfrac{x_n}{x_{n-1}}$ )
$S' = (n+1)$ $-$ ( $\dfrac{x_1}{x_0} + \dfrac{x_2}{x_1} + ... + \dfrac{x_i}{x_{i-1}} + \dfrac{k}{x_i} + \dfrac{x_{i+1}}{k}+\dfrac{x_{i+2}}{x_{i+1}} +... + \dfrac{x_n}{x_{n-1}}$ )
Ta có $S' - S $=$ \dfrac{ x_{i+1}}{x_i} - \dfrac{x_{i+1}}{k} - \dfrac{k}{x_i} +1 =\dfrac{(x_i - k )( k - x_{i+1})}{k . x_i} $ $> 0$
Chèn k vào giữa x_i và x_{i+1} và được bộ số mới làm cho tổng S đạt giá trị lớn hơn.
Tương tự đến khi không còn số tự nhiên k nào thỏa mãn điều kiện trên thì tổng S đạt giá trị lớn nhất.
Điều trên có nghĩa tổng S có giá trị lớn nhất là $\dfrac{1}{2} + \dfrac{1}{3} + ... + \dfrac{1}{ 100} $và bộ số của chúng ta là ( $ 99, 98, ..., 1 $)

Lưu ý:
Trên đây chỉ là mình viết ra suy nghĩ nên rất lộn xộn, ko phải là 1 lời giải đầy đủ.

Bài viết đã được chỉnh sửa nội dung bởi inhtoan: 23-04-2009 - 17:32

Rất hân hạnh được làm quen và học cùng với các anh chị và các bạn!
Mong được sự giúp đỡ của tất cả mọi người!




1 người đang xem chủ đề

0 thành viên, 1 khách, 0 thành viên ẩn danh